Die Lagrangedichte eines freien Teilchens in Landau & Lifshitz

In Landau & Lifshitz' Herleitung der Lagrangefunktion eines freien Teilchens in einem galiläischen Bezugssystem findet man folgendes Argument: Die Bewegungsgleichungen in zwei galiläischen Bezugssystemen müssen identisch sein; daher müssen sich die jeweiligen Lagrange-Operatoren durch die Gesamtableitung einer Funktion der verallgemeinerten Position und der Zeit unterscheiden. Dies ist im Wesentlichen das Gegenteil dessen, worauf die Autoren als Rechtfertigung hinweisen, nämlich dass das Hinzufügen eines solchen Begriffs zur Lagrange-Funktion die Gleichungen unverändert lässt, und ich verstehe nicht wirklich, warum dies gilt. Die einzige relevante Antwort, die ich auf Stackexchange gefunden habe, ist Qmechanics' Take in Deriving the Lagrangian for a free Particle , aber ich muss zugeben, dass es mich nicht ganz zufrieden stellt.

Bearbeiten: Ich frage, warum Modifikationen des Lagrange, die die EL-Gleichungen nicht ändern, notwendigerweise die Addition einer Gesamtableitung (und Multiplikation mit einem Skalar) sind, wie L & L-Behauptungen.

Ich könnte mich irren, aber die Verwendung der Euler-Lagrange-Gleichungen in diesem Zusammenhang ist ziemlich zirkulär, oder? Was wir beweisen sollten, ist, dass etwas, das die Gleichungen nicht ändert, eine totale Ableitung usw. ist, nicht, dass etwas, das die Gleichungen nicht ändert, die Gleichungen nicht ändert. Auch die Antwort, die ich verlinkt habe, gibt die endgültige Form des Lagrangian an, während Landau nach dem von mir erwähnten ein anderes Argument verwendet.
Beachten Sie, dass meine Verwendung von EL eqs. ist keine zirkuläre Logik. Es ist ein Trick, Terme zu identifizieren, die totale Ableitungen sind.
Ich muss zugeben, das ist etwas verschwommen, aber ich nehme an, Sie korrigieren nicht Landaus Methode, sondern Ihre ist völlig anders; er sagt, dass sich die Lagrange-Operatoren durch eine bestimmte Art von Funktion unterscheiden müssen, und stellt dann fest, dass dL/d(v^2) eine Konstante ist, aber Sie sagen, dass die Differenz Euler-Lagrange gehorchen muss, und dann finden Sie eine Formel dafür; oder verstehe ich das falsch?
FWIW, L&L scheinen die Multiplikation mit einem Skalar nicht als OP-Behauptungen (v2) zu erwähnen.
Beide Schritte, die Sie verlinkt haben, habe ich nicht die Voraussetzungen, um sie gut zu verstehen. Was die Multiplikation mit einem Skalar betrifft, schließt Landau dies in Absatz 3 aus (das heißt, im Zusammenhang mit zwei Systemen). Entschuldigung, wenn ich etwas unverblümt bin, ich verstehe nicht, warum es keine sofort verfügbare Antwort auf meine Frage gibt; sollte es schließlich nicht jeden etwas mathematisch veranlagten Landau-Leser verfolgen? PS: Ich denke, meine Frage verdient das Tag "unklar" nicht.
Wenn ich hinzufügen darf (und natürlich mit gebührendem Respekt vor Landau!), ist die ganze Herleitung voll von logischen Fehlern. Wenn man herausgefunden hat, dass L eine Konstante multipliziert mit der Geschwindigkeit im Quadrat ist, scheinen die Autoren zu verwenden, um zu beweisen, dass die Konstante positiv ist, dass es einen eindeutigen Pfad gibt, der die Wirkung minimiert, ohne die Tatsache zu beweisen oder auch nur zu erwähnen.
OK. Das ist Gl. (2,7) Zoll § 2 . Link: renaissance.ucsd.edu/courses/mae207/mech.pdf

Antworten (1)

Die Logik von L&L ist wie folgt:

  1. L&L-Anforderungen 1 dass eine (infinitesimale) Galilei-Transformation eine Quasisymmetrie (QS) des gesuchten Wirkungsfunktionals sein sollte S .

  2. Das fordern wir außerdem

    • das Aktionsfunktional ist lokal, und
    • der Ortsraum ist kontrahierbar.
  3. Aus diesem Phys.SE-Beitrag leiten wir dann ab, dass eine (infinitesimale) Galilei-Transformation tatsächlich eine QS der Lagrange- Transformation ist L selbst. Dies bedeutet per Definition, dass die Änderung Δ L im Lagrange ist eine Gesamtzeitableitung, wie OP zeigen wollte.

--

1 Dies ist vernünftig, da die Newtonsche Mechanik eine Galileische Symmetrie hat. Es gibt jedoch möglicherweise ein Schlupfloch, da eine Symmetrie von EOM kein QS der Aktion sein muss, vgl. zB dieser Phys.SE Beitrag. Dann stellt sich die größere Frage:

Wie stark können wir die Aktion ändern, ohne den EOM zu beeinflussen?

Das ist eine gute Frage, die im Wesentlichen auch in diesem Phys.SE-Beitrag gestellt wurde .